18.07.2014 Views

Topics in Inequalities - Theorems and Techniques Hojoo ... - Index of

Topics in Inequalities - Theorems and Techniques Hojoo ... - Index of

Topics in Inequalities - Theorems and Techniques Hojoo ... - Index of

SHOW MORE
SHOW LESS

Create successful ePaper yourself

Turn your PDF publications into a flip-book with our unique Google optimized e-Paper software.

Corollary 1.2.1. Let p, q, <strong>and</strong> r be positive real numbers. Let θ 1 , θ 2 , <strong>and</strong> θ 3 be real numbers satisfy<strong>in</strong>g<br />

θ 1 + θ 2 + θ 3 = π. Then, the follow<strong>in</strong>g <strong>in</strong>equality holds.<br />

p cos θ 1 + q cos θ 2 + r cos θ 3 ≤ 1 ( qr<br />

2 p + rp<br />

q + pq )<br />

.<br />

r<br />

(√ √<br />

Pro<strong>of</strong>. Take (x, y, z) = qr<br />

p , rp<br />

q , √ )<br />

pq<br />

r<br />

<strong>and</strong> apply the above proposition.<br />

Theorem 1.2.2. (Barrow’s Inequality) Let P be an <strong>in</strong>terior po<strong>in</strong>t <strong>of</strong> a triangle ABC <strong>and</strong> let U, V , W<br />

be the po<strong>in</strong>ts where the bisectors <strong>of</strong> angles BP C, CP A, AP B cut the sides BC,CA,AB respectively. Prove<br />

that P A + P B + P C ≥ 2(P U + P V + P W ).<br />

Pro<strong>of</strong>. ([MB] <strong>and</strong> [AK]) Let d 1 = P A, d 2 = P B, d 3 = P C, l 1 = P U, l 2 = P V , l 3 = P W , 2θ 1 = ∠BP C,<br />

2θ 2 = ∠CP A, <strong>and</strong> 2θ 3 = ∠AP B. We need to show that d 1 + d 2 + d 3 ≥ 2(l 1 + l 2 + l 3 ). It’s easy to deduce<br />

the follow<strong>in</strong>g identities<br />

l 1 = 2d 2d 3<br />

d 2 + d 3<br />

cos θ 1 , l 2 = 2d 3d 1<br />

d 3 + d 1<br />

cos θ 2 , <strong>and</strong> l 3 = 2d 1d 2<br />

d 1 + d 2<br />

cos θ 3 ,<br />

By the AM-GM <strong>in</strong>equality <strong>and</strong> the above corollary, this means that<br />

l 1 + l 2 + l 3 ≤ √ d 2 d 3 cos θ 1 + √ d 3 d 1 cos θ 2 + √ d 1 d 2 cos θ 3 ≤ 1 2 (d 1 + d 2 + d 3 ) .<br />

As another application <strong>of</strong> the above trigonometric proposition, we establish the follow<strong>in</strong>g <strong>in</strong>equality<br />

Corollary 1.2.2. ([AK], Abi-Khuzam) Let x 1 , · · · , x 4 be positive real numbers. Let θ 1 , · · · , θ 4 be real<br />

numbers such that θ 1 + · · · + θ 4 = π. Then,<br />

√<br />

(x 1 x 2 + x 3 x 4 )(x 1 x 3 + x 2 x 4 )(x 1 x 4 + x 2 x 3 )<br />

x 1 cos θ 1 + x 2 cos θ 2 + x 3 cos θ 3 + x 4 cos θ 4 ≤<br />

.<br />

x 1 x 2 x 3 x 4<br />

√<br />

Pro<strong>of</strong>. Let p = x12 2<br />

+x 2<br />

2x 1 x 2<br />

+ x32 2<br />

+x 4<br />

2x 3 x 4<br />

q = x1x2+x3x4<br />

2<br />

<strong>and</strong> λ = p<br />

q . In the view <strong>of</strong> θ 1 + θ 2 + (θ 3 + θ 4 ) = π <strong>and</strong><br />

θ 3 + θ 4 + (θ 1 + θ 2 ) = π, the proposition implies that<br />

x 1 cos θ 1 + x 2 cos θ 2 + λ cos(θ 3 + θ 4 ) ≤ pλ = √ pq,<br />

<strong>and</strong><br />

x 3 cos θ 3 + x 4 cos θ 4 + λ cos(θ 1 + θ 2 ) ≤ q λ = √ pq.<br />

S<strong>in</strong>ce cos(θ 3 + θ 4 ) + cos(θ 1 + θ 2 ) = 0, add<strong>in</strong>g these two above <strong>in</strong>equalities yields<br />

√<br />

x 1 cos θ 1 + x 2 cos θ 2 + x 3 cos θ 3 + x 4 cos θ 4 ≤ 2 √ (x 1 x 2 + x 3 x 4 )(x 1 x 3 + x 2 x 4 )(x 1 x 4 + x 2 x 3 )<br />

pq =<br />

.<br />

x 1 x 2 x 3 x 4<br />

10

Hooray! Your file is uploaded and ready to be published.

Saved successfully!

Ooh no, something went wrong!